43
$\begingroup$

In an answer to the popular question on common false beliefs in mathematics

Examples of common false beliefs in mathematics

I mentioned that many people conflate the two different kinds of formal Laurent series field in two variables. Let $K$ be any field. Then we have the joint Laurent series field, the field of fractions

$K((x,y)) = \operatorname{Frac}(K[[x,y]])$

and also the iterated Laurent series field

$K((x))((y)) = \left(K((x))\right)((y))$.

These fields are not the same, roughly because when we write out an arbitrary element of the iterated field as a formal Laurent series in $y$, for each (say non-negative) $n$, the coefficient of $y^n$ is allowed to be an arbitrary formal Laurent series in $x$. In particular, as $n$ varies, arbitrarily large negative powers of $x$ may appear.

However, this is rather far from a convincing argument. Indeed, I gave the following explicit (and fallacious!) example: $\sum_{n=0}^{\infty} x^{-n} y^n$. But in a comment to my answer, user AS points out that this element is equal (in the iterated field, say) to $\frac{1}{1-\frac{y}{x}}$ and therefore it must lie in the fraction field of $K[[x,y]]$. Evidently the fallacy here is that the fraction field of $K[[x,y]]$ is the field of all formal Laurent series which are finite-tailed in both $x$ and $y$. But as this example shows, the latter isn't even a field, unlike the one-variable case.

[At least when $K = \mathbb{C}$, by less explicit means one can see that these two fields are very different: e.g., the joint field is Hilbertian so has nonabelian Galois group, whereas the iterated field has Galois group $\widehat{\mathbb{Z}}^2$.]

AS offered to write down, with proof, an explicit element of $K((x))((y)) \setminus K((x,y))$, so I decided to post a question asking for such a guy. Of course, there is more than one such element -- or better put, more than one type of construction of such elements -- so I would be interested to see multiple answers to:

Please exhibit, with proof, an explicit element of $K((x))((y)) \setminus K((x,y))$.

$\endgroup$
4
  • 8
    $\begingroup$ I'd also like to add to the question: how can this difference be visualized in terms of formal neighborhoods? K[[x,y]] is supposedly the formal neighborhood of (0,0) in an affine plane; and K((x,y))=K[[x,y]][x^(-1),y^(-1)], so that can be viewed as looking outside of the closed fiber. How does K((x))((y)) relate to this? $\endgroup$ Jul 31, 2010 at 14:23
  • 5
    $\begingroup$ I think that the purported formula K((x,y))=K[[x,y]][x^(-1),y^(-1)] extends the fallacious tradition: (x-y)^(-1) is certainly in the fraction field of K[[x,y]], however, it is not in the localization of K[[x,y]] with respect to xy. $\endgroup$ Jul 31, 2010 at 18:10
  • 1
    $\begingroup$ Also, I wonder if there is a symmetry argument to distinguish K((x,y)), as included in K((x))((y)), from K((x))((y)) itself. The first has an involutive ring automorphism exchanging x with y. Can one show that nothing similar may possibly exist on K((x))((y))? $\endgroup$ Aug 1, 2010 at 16:26
  • $\begingroup$ So I see that I came way too late to answer this question (as Pete suggested). But that's fine, since several great answers have been given! I was thinking along the same lines as Tony Scholl, mentioning the explicit formula Sum(y^n/x^{n^2}) in the original thread. $\endgroup$
    – Wanderer
    Aug 1, 2010 at 21:59

5 Answers 5

15
$\begingroup$

Suppose $R$ is a domain with field of fractions $F$. Let $f\in F[[y]]$ and suppose that $f\in Frac(R[[y]])$. Then $f=h/g$ with $g,h\in R[[y]]]$ and we may assume that $g=b_0+b_1y+\dots$ with $b_0\ne0$. Therefore $$ b_0g^{-1}=(1+(b_1/b_0)y+\dots)^{-1}\in R[[y/b_0]] $$ and so $f\in b_0^{-1}R[[y/b_0]]$.

So when $R=k[[x]]$, any $f=\sum y^n/x^{r(n)}$ with $r(n)/n \to \infty$ will work.

An analytic paraphrase of this argument (which is relevant to Makhalan's comment) is that (regarding $F=k((x))$ as a valued field) any element of $Frac(k[[x,y]])$ is a ratio of power series converging on the open unit disc in $F$. So it is enough to write down a power series in $y$ with zero radius of convergence.

$\endgroup$
5
  • 1
    $\begingroup$ That's a very nice way to look at it! $\endgroup$ Jul 31, 2010 at 15:35
  • 3
    $\begingroup$ The example appears to do the job, but one needs to be cautious with notations such as $R[[y/b_0]]:$ is this supposed to be a subring of some larger ring? One of the difficulties in dealing with iterated formal power/Laurent series rings is that "obvious" embeddings are given by certain series expansion maps and compatibility of these embeddings (given by composing expansions) requires a proof. An illustration: the image of $(x-y)^{-1}$ under the embedding $K(x,y)\subset K((x))((y))$ is computed as $$(x-y)^{-1}=x^{-1}(1-y/x)^{-1}=\sum_{n\geq 0}y^n x^{-n-1}.$$ $\endgroup$ Jul 31, 2010 at 19:10
  • 2
    $\begingroup$ Interchanging $x$ and $y$, we get an expansion $$(x-y)^{-1}=-\sum_{n\geq 0}x^n y^{-n-1}$$ in the embedding $K(x,y)\subset K((y))((x))$ incompatible with the previous one. (This is related with some algebraic properties of iterated residues that play role in the foundations of vertex operator algebras, for example.) $\endgroup$ Jul 31, 2010 at 19:19
  • $\begingroup$ Victor: a very good point. Laurent series over Laurent series need double "dangerous bends" - as you have just pointed out, $k((x))((y))$ and $k((y))((x))$ are entirely different rings. (This sort of issue also comes up trying to relate all of Fontaine's $B_?$ rings which occur in the study of $p$-adic representations). But here, everything can live in $F((y))$. $\endgroup$ Jul 31, 2010 at 19:31
  • $\begingroup$ Another way to say it is, consider the subfield of K((x))((y)), consisting of all Laurent formal series $\sum_k c_k(x)y^k$ whose coefficients sequence $c_k\in K((x))$ satisfies $\inf_k \frac{ord(c_k)}{k} > -\infty$. It contains $K[[x,y]]$, and it's of course a proper subfield. $\endgroup$ Aug 1, 2010 at 18:14
8
$\begingroup$

There is a characterization of $K((x,y))$ which makes it clear what sorts of examples to expect. That is, it is a subring of $K(z)((y))$ (where $x = yz$) of series with certain regularity properties.

Let $R \subset K[z][[y]]$ be the "filtered power series" ring, consisting of series $p(z,y) = \sum_{n \geq 0} p_n(z) y^n$ with $\deg p_n \leq n$; equivalently, $p_n(x/y) y^n$ is homogeneous in $x,y$ of degree $n$. Then by definition the substitution $x = yz$ furnishes an isomorphism $R \cong K[[x,y]]$. One verifies that $R$ is indeed a ring and that if $p(z,0) = 1$, then $p$ is a unit in $R$, since $$p(z,y)^{-1} = (1 + \sum_{n \geq 1} p_n(z) y^n)^{-1} = \sum_{m \geq 0} (-1)^m (\sum_{n \geq 1} p_n(z) y^n)^m.$$

Now we consider $\mathrm{Frac}(R)$. If $p(z,y) = \sum_{n \geq a} p_n(z) y^n$ with $p_a \neq 0$, then we can write $$p(z,y) = p_a(z) y^a q(z,y)$$ in $K(z)[[y]]$, where if $q(z,y) = \sum_{n \geq 0} q_n(z) y^n$, then $q_n(x/y) y^n$ is homogeneous of degree $n$ (as a rational function). Then $p^{-1} = p_a^{-1} y^{-a} q^{-1} \in K(z)((y))$ also has this property.

Conversely, if $\ell(z,y) \in K(z)((y))$ is a "filtered Laurent series": $\ell(z,y) = \sum_{n \geq -a} \ell_n(z) y^n$ with $a \in \mathbb{Z}$, $\ell_n(z) \in K(z)$, and $\ell_n(x/y)y^n$ homogeneous of degree $n \in \mathbb{Z}$, then $\ell(x/y,y)$ makes sense in $K((x,y))$ if each $\ell_n$ is additionally of the form $q_{n + a}(z) p_a(z)^{-1}$ for polynomials $q_{n + a}, p_a$ of degrees at most $n + a, a$ respectively, since then $$\ell(x/y,y) = p_a(x/y)^{-1} y^{-a} \sum_{m \geq 0} q_m(x/y) y^m,$$ with $p_a(x/y)^{-1} \in K(x/y) \subset K((x,y))$ and $q_m(x/y) y^m \in K[x,y]$ homogeneous of degree $m$. More concisely, we can say: $K((x,y))$ is isomorphic to the subring of $K(z)((y))$ consisting of series $$\ell(z,y) = \sum_{n \geq -a} \ell_n(z) y^n$$ such that each $\ell_n(x/y)y^n$ is homogeneous of degree $n$ and all the $\ell_n(z)$ together have only finitely many poles in $\bar{K}$.

Thus, the following classes of functions in $K((x))((y))$ are not in $K((x,y))$:

  • Series of the form $\ell(x,y) = t(x/y) = \sum_{n \geq 0} t_n x^n y^{-n}$ for which $t(z) \in K[[z]]$ is not a rational function. For example, $$t(z) = \exp(z) = \sum_{n \geq 0} \frac{1}{n!} z^n, \qquad t(z) = \sum_{n \geq 0} b_n z^n,$$ where the first only makes sense over a field of characteristic zero, and where in the second, $b_n \in \{0,1\}$ is a random sequence of bits (this will almost surely not be rational). More generally, series of the form $\ell(x,y) = \sum_{a,b} \ell_{a,b} x^a y^b$ such that some $\ell_n(z) = \sum_{a + b = n} \ell_{a,b} z^a$ is not rational.
  • Series $\ell(x,y)$ for which the $\ell_n(z)$ have "infinitely many poles" among them. This is hard to detect on the level of formal series, but one criterion is similar to that of Tony Scholl: the radii of convergence of the $\ell_n(z)$ have infimum zero or infinity. Over $\mathbb{C}$, by the root test this is the same as $\lim_{n \to \infty} \sqrt[n]{|\ell_{a, n - a}|}$ being unbounded in $a$, or their inverses being unbounded. (This also works over any field with a complete real valuation.)
$\endgroup$
9
  • $\begingroup$ I like this answer, but I suggest the following replacement: Laurent polynomial -> Laurent series. $\endgroup$
    – S. Carnahan
    Jul 31, 2010 at 18:12
  • $\begingroup$ Also, lowest degree -> lowest total degree? $\endgroup$
    – S. Carnahan
    Jul 31, 2010 at 18:14
  • $\begingroup$ I cannot follow this explanation: are your $p_0(x,y)$ and $q_0(x,y)$ meant to be elements of $K[[x,y]]$ or its extension? Note that in $K[[x,y]]$ it may be impossible to divide an element by its lowest degree homogeneous part, because the Newton polygon of a product is contained in the Newton polygons of factors. An easy explicit example is $p=x+y^2.$ (Here the Newton polygon is obtained from the convex hull of the exponents of all nonzero monomials by adding $\mathbb{Z}_+^2$.) $\endgroup$ Jul 31, 2010 at 18:35
  • $\begingroup$ They are in $K((x,y))$, but also in $K(x/y)[[y]]$, and it is in the latter ring that they are invertible. In your example, $f(x,y) = x$ and $f_0(x/y) = x/y$; then $p_0(x,y) = 1 + x^{-1} y^2 = 1 + (x/y)^{-1} y$, which is invertible as a power series in $y$ having rational-function coefficients. $\endgroup$
    – Ryan Reich
    Jul 31, 2010 at 18:53
  • $\begingroup$ Victor: the easiest way to see this is to substitute $x=yz$ into an element of $k[[x,y]]$. In the new power series, the coefficient of $y^n$ is a polynomial in $z$ of degree at most $n$, so $k[[x,y]] \subset k[x/y][[y]]$. $\endgroup$ Jul 31, 2010 at 19:03
6
$\begingroup$

Addendum: actually there is a chain of subfields between $K((x,y))$ and $K((x))((y))$ with cardinality c. For instance, for any $\lambda>1$ we may consider the subset $R_\lambda$ of $K((x))((y))$ of all Laurent series $\sum_k c_k(x)y^k$ with $c_k\in K((x))$ satisfying $$\inf_k \lambda ^{-k} \mathrm{ord}(c_k) > -\infty.$$ It's easy to check that it's a subfield of $K((x))((y))$, containing $K[[x,y]]$.

Moreover, since in place of $\lambda^k$ we can use functions $\mathbb{N}\to\mathbb{N}$ with arbitrarily large growth, one can also show that the subfields between $K((x,y))$ and $K((x))((y))$ have uncountable cofinality also.

$\endgroup$
1
  • 1
    $\begingroup$ You have made this question so delightfully absurd I feel obligated to vote you up :) $\endgroup$
    – Ryan Reich
    Aug 2, 2010 at 3:55
5
$\begingroup$

I think that $\displaystyle\sum\limits_{n\geq 0}\frac{y^{2^n}}{x^{2^n}}$ is such a case. In fact, would it lie in $K\left(\left(x,y\right)\right)$, we would have two nonzero power series $\sum\limits_{u,v\geq 0}a_{u,v}x^uy^v$ and $\sum\limits_{u,v\geq 0}b_{u,v}x^uy^v$ satisfying $\displaystyle\sum\limits_{n\geq 0}\frac{y^{2^n}}{x^{2^n}}\cdot \sum\limits_{u,v\geq 0}a_{u,v}x^uy^v=\sum\limits_{u,v\geq 0}b_{u,v}x^uy^v$. This rewrites coefficientwise as

(1) $\displaystyle b_{u,v}=\sum\limits_{n\geq 0}a_{u+2^n,v-2^n}$

for all $u$ and $v$ (maybe the $+$ and the $-$ signs are the other way round here, but I hope it is clear what to do then), where $a_{p,q}$ is defined to be $0$ if $p<0$ or $q<0$. Now, take some $\left(p,q\right)$ satisfying $a_{p,q}\neq 0$, and take some natural $N$ which is way bigger than $p$ and $q$. Apply (1) to $u=p-2^N$ and $v=q+2^N$ and obtain $b_{p-2^N,q+2^N}=a_{p,q}$ (because in the sum on the right hand side of (1), the summands for $n < N$ have $u+2^n<0$ and thus are zero, while the summands for $n > N$ have $v-2^n<0$ and thus are zero as well, so only the $n=N$ summand remains), contradicting $p-2^N<0$ (which is because $N$ is so huge).

$\endgroup$
4
$\begingroup$

This really should be a comment to Tony Scholl's nice counterexample above, but I dont have the necessary reputation. (!!!)

A transparent way to see that any $\sum_{n\ge 0} \dfrac{y^n}{x^{r(n)}}$ with $\dfrac{r(n)}{n}$ tending to $\infty$ will serve as a counterexample (without dividing by $b_0$, etc.) is as follows: More generally, suppose $\sum_{k \ge 0} a_k(x) y^k$ is to be a counterexample, with $a_k(x)$ coming from $K((x))$ ($K$ will be the base field), and $v_x(a_k(x)):= -r(k) < 0)$ ($v_x$ being the $x$-adic valuation on $K((x))$ corresponding to the lowest degree term in $x$.) We wish to show that it is impossible to write it as the quotient $\sum_{i \ge 0} p_i(x) y^i$ divided by $\sum_{j \ge 0} q_j(x) y^j$, where, importantly, each $v_x(p_i(x))$ and $v_x(q_j(x))$ are non-negative, and $q_0(x)\neq 0$. (Note that we are embedding $K[[x,y]]$ in $K((x))((y))$ by writing elements of $K[[x,y]]$ as a power series in $y$ with coeffs in $K[[x]]$, see the discussions on this above!) Then cross-multiplying and comparing coeffs of $y^k$ we get $$\sum_{j=0}^k a_{k-j}(x)q_{j}(x) = p_k(x).$$ If $v_x(q_j(x)) := d_j \ge 0$, then the nonzero summands on the left have $v_x$ (i.e., exponent of lowest $x$-terms) $-r(k) + d_0$, $-r(k-1)+d_1$, etc. The lowest $x$-term on the right is of non-negative exponent. To make this an impossibility (so that we get a counterexample), it is sufficient to choose the function $r$ to satisfy $-r(k) + d_0 < -r(k-1)$ for any choice of $d_0\ge 0$ and for all $k > k_0(d_0)$. (Recall that the $r(k)$ are negative integers while the d_i are non-negative integers. This condition makes the $r(k)$ increase monotonically, and dominate all $r(j)$ after a while.) We write this as $r(k) - r(k-1) > d_0$ for any $d_0\ge 0$, for all $k > k_0(d_0)$. Writing this as $r(n) > r(k_0) +(n-k_0)d_0$ for $n > k_0(d_0)$ and dividing by $n$, we find that $\dfrac{r(n)}{n}$ goes to $\infty$. Pietro Majer's comment also follows from this.

$\endgroup$

Your Answer

By clicking “Post Your Answer”, you agree to our terms of service and acknowledge you have read our privacy policy.

Not the answer you're looking for? Browse other questions tagged or ask your own question.